Exponents of ec primes ending with digit 7











up vote
0
down vote

favorite












A conjecture about numbers of the form $10^{m}(2^{k}−1)+2^{k-1}−1$, where $m$ is the number of decimal digits of $ 2^{k-1}$.
In this question ec-numbers are introduced, obttained by the concatenation of two consecutive Mersenne numbers (12763=ec(7) for example). The values of $k$ for which $ec(k)$ is prime are: $[2, 3, 4, 7, 8, 12, 19, 22, 36, 46, 51, 67, 79, 215, 359, 394, 451, 1323, 2131, 3336, 3371, 6231, 19179, 39699, 51456, 56238, 69660, 75894, 79798, 92020, 174968, 176006, 181015, 285019, 331259, 360787, 366770]$
$7, 67, 360787$ are all the $k's$ leading to a prime ending with digit $7$. $7,67,360787$ are all congruent to $1$ $pmod 6$. My question is: does a k exist such that k ends with digit $7$, $ec(k)$ is prime and $k$ is NOT congruent to 1 $pmod 6$? Or that can be ruled out? In other words, if $k$ is congruent to 7 mod 10 and $ec(k)$ is prime, then necessarly $k$ must be congruent to 1 mod 6?










share|cite|improve this question









New contributor




paolo galli is a new contributor to this site. Take care in asking for clarification, commenting, and answering.
Check out our Code of Conduct.




















  • I noticed that the largest factor of $6,66,360786$ is a palindrome. The largest factor of 6 is 3, the largest factor of 66 is 11, the largest factor of 360786 is 383.
    – paolo galli
    6 hours ago










  • @Giovanni Resta can you find a counter-example?
    – paolo galli
    6 hours ago










  • @Peter i think it is hard to find a counterexample, maybe it can be ruled out with a proof.
    – paolo galli
    6 hours ago















up vote
0
down vote

favorite












A conjecture about numbers of the form $10^{m}(2^{k}−1)+2^{k-1}−1$, where $m$ is the number of decimal digits of $ 2^{k-1}$.
In this question ec-numbers are introduced, obttained by the concatenation of two consecutive Mersenne numbers (12763=ec(7) for example). The values of $k$ for which $ec(k)$ is prime are: $[2, 3, 4, 7, 8, 12, 19, 22, 36, 46, 51, 67, 79, 215, 359, 394, 451, 1323, 2131, 3336, 3371, 6231, 19179, 39699, 51456, 56238, 69660, 75894, 79798, 92020, 174968, 176006, 181015, 285019, 331259, 360787, 366770]$
$7, 67, 360787$ are all the $k's$ leading to a prime ending with digit $7$. $7,67,360787$ are all congruent to $1$ $pmod 6$. My question is: does a k exist such that k ends with digit $7$, $ec(k)$ is prime and $k$ is NOT congruent to 1 $pmod 6$? Or that can be ruled out? In other words, if $k$ is congruent to 7 mod 10 and $ec(k)$ is prime, then necessarly $k$ must be congruent to 1 mod 6?










share|cite|improve this question









New contributor




paolo galli is a new contributor to this site. Take care in asking for clarification, commenting, and answering.
Check out our Code of Conduct.




















  • I noticed that the largest factor of $6,66,360786$ is a palindrome. The largest factor of 6 is 3, the largest factor of 66 is 11, the largest factor of 360786 is 383.
    – paolo galli
    6 hours ago










  • @Giovanni Resta can you find a counter-example?
    – paolo galli
    6 hours ago










  • @Peter i think it is hard to find a counterexample, maybe it can be ruled out with a proof.
    – paolo galli
    6 hours ago













up vote
0
down vote

favorite









up vote
0
down vote

favorite











A conjecture about numbers of the form $10^{m}(2^{k}−1)+2^{k-1}−1$, where $m$ is the number of decimal digits of $ 2^{k-1}$.
In this question ec-numbers are introduced, obttained by the concatenation of two consecutive Mersenne numbers (12763=ec(7) for example). The values of $k$ for which $ec(k)$ is prime are: $[2, 3, 4, 7, 8, 12, 19, 22, 36, 46, 51, 67, 79, 215, 359, 394, 451, 1323, 2131, 3336, 3371, 6231, 19179, 39699, 51456, 56238, 69660, 75894, 79798, 92020, 174968, 176006, 181015, 285019, 331259, 360787, 366770]$
$7, 67, 360787$ are all the $k's$ leading to a prime ending with digit $7$. $7,67,360787$ are all congruent to $1$ $pmod 6$. My question is: does a k exist such that k ends with digit $7$, $ec(k)$ is prime and $k$ is NOT congruent to 1 $pmod 6$? Or that can be ruled out? In other words, if $k$ is congruent to 7 mod 10 and $ec(k)$ is prime, then necessarly $k$ must be congruent to 1 mod 6?










share|cite|improve this question









New contributor




paolo galli is a new contributor to this site. Take care in asking for clarification, commenting, and answering.
Check out our Code of Conduct.











A conjecture about numbers of the form $10^{m}(2^{k}−1)+2^{k-1}−1$, where $m$ is the number of decimal digits of $ 2^{k-1}$.
In this question ec-numbers are introduced, obttained by the concatenation of two consecutive Mersenne numbers (12763=ec(7) for example). The values of $k$ for which $ec(k)$ is prime are: $[2, 3, 4, 7, 8, 12, 19, 22, 36, 46, 51, 67, 79, 215, 359, 394, 451, 1323, 2131, 3336, 3371, 6231, 19179, 39699, 51456, 56238, 69660, 75894, 79798, 92020, 174968, 176006, 181015, 285019, 331259, 360787, 366770]$
$7, 67, 360787$ are all the $k's$ leading to a prime ending with digit $7$. $7,67,360787$ are all congruent to $1$ $pmod 6$. My question is: does a k exist such that k ends with digit $7$, $ec(k)$ is prime and $k$ is NOT congruent to 1 $pmod 6$? Or that can be ruled out? In other words, if $k$ is congruent to 7 mod 10 and $ec(k)$ is prime, then necessarly $k$ must be congruent to 1 mod 6?







number-theory






share|cite|improve this question









New contributor




paolo galli is a new contributor to this site. Take care in asking for clarification, commenting, and answering.
Check out our Code of Conduct.











share|cite|improve this question









New contributor




paolo galli is a new contributor to this site. Take care in asking for clarification, commenting, and answering.
Check out our Code of Conduct.









share|cite|improve this question




share|cite|improve this question








edited 7 hours ago





















New contributor




paolo galli is a new contributor to this site. Take care in asking for clarification, commenting, and answering.
Check out our Code of Conduct.









asked 7 hours ago









paolo galli

163




163




New contributor




paolo galli is a new contributor to this site. Take care in asking for clarification, commenting, and answering.
Check out our Code of Conduct.





New contributor





paolo galli is a new contributor to this site. Take care in asking for clarification, commenting, and answering.
Check out our Code of Conduct.






paolo galli is a new contributor to this site. Take care in asking for clarification, commenting, and answering.
Check out our Code of Conduct.












  • I noticed that the largest factor of $6,66,360786$ is a palindrome. The largest factor of 6 is 3, the largest factor of 66 is 11, the largest factor of 360786 is 383.
    – paolo galli
    6 hours ago










  • @Giovanni Resta can you find a counter-example?
    – paolo galli
    6 hours ago










  • @Peter i think it is hard to find a counterexample, maybe it can be ruled out with a proof.
    – paolo galli
    6 hours ago


















  • I noticed that the largest factor of $6,66,360786$ is a palindrome. The largest factor of 6 is 3, the largest factor of 66 is 11, the largest factor of 360786 is 383.
    – paolo galli
    6 hours ago










  • @Giovanni Resta can you find a counter-example?
    – paolo galli
    6 hours ago










  • @Peter i think it is hard to find a counterexample, maybe it can be ruled out with a proof.
    – paolo galli
    6 hours ago
















I noticed that the largest factor of $6,66,360786$ is a palindrome. The largest factor of 6 is 3, the largest factor of 66 is 11, the largest factor of 360786 is 383.
– paolo galli
6 hours ago




I noticed that the largest factor of $6,66,360786$ is a palindrome. The largest factor of 6 is 3, the largest factor of 66 is 11, the largest factor of 360786 is 383.
– paolo galli
6 hours ago












@Giovanni Resta can you find a counter-example?
– paolo galli
6 hours ago




@Giovanni Resta can you find a counter-example?
– paolo galli
6 hours ago












@Peter i think it is hard to find a counterexample, maybe it can be ruled out with a proof.
– paolo galli
6 hours ago




@Peter i think it is hard to find a counterexample, maybe it can be ruled out with a proof.
– paolo galli
6 hours ago















active

oldest

votes











Your Answer





StackExchange.ifUsing("editor", function () {
return StackExchange.using("mathjaxEditing", function () {
StackExchange.MarkdownEditor.creationCallbacks.add(function (editor, postfix) {
StackExchange.mathjaxEditing.prepareWmdForMathJax(editor, postfix, [["$", "$"], ["\\(","\\)"]]);
});
});
}, "mathjax-editing");

StackExchange.ready(function() {
var channelOptions = {
tags: "".split(" "),
id: "69"
};
initTagRenderer("".split(" "), "".split(" "), channelOptions);

StackExchange.using("externalEditor", function() {
// Have to fire editor after snippets, if snippets enabled
if (StackExchange.settings.snippets.snippetsEnabled) {
StackExchange.using("snippets", function() {
createEditor();
});
}
else {
createEditor();
}
});

function createEditor() {
StackExchange.prepareEditor({
heartbeatType: 'answer',
convertImagesToLinks: true,
noModals: true,
showLowRepImageUploadWarning: true,
reputationToPostImages: 10,
bindNavPrevention: true,
postfix: "",
imageUploader: {
brandingHtml: "Powered by u003ca class="icon-imgur-white" href="https://imgur.com/"u003eu003c/au003e",
contentPolicyHtml: "User contributions licensed under u003ca href="https://creativecommons.org/licenses/by-sa/3.0/"u003ecc by-sa 3.0 with attribution requiredu003c/au003e u003ca href="https://stackoverflow.com/legal/content-policy"u003e(content policy)u003c/au003e",
allowUrls: true
},
noCode: true, onDemand: true,
discardSelector: ".discard-answer"
,immediatelyShowMarkdownHelp:true
});


}
});






paolo galli is a new contributor. Be nice, and check out our Code of Conduct.










 

draft saved


draft discarded


















StackExchange.ready(
function () {
StackExchange.openid.initPostLogin('.new-post-login', 'https%3a%2f%2fmath.stackexchange.com%2fquestions%2f3004600%2fexponents-of-ec-primes-ending-with-digit-7%23new-answer', 'question_page');
}
);

Post as a guest















Required, but never shown






























active

oldest

votes













active

oldest

votes









active

oldest

votes






active

oldest

votes








paolo galli is a new contributor. Be nice, and check out our Code of Conduct.










 

draft saved


draft discarded


















paolo galli is a new contributor. Be nice, and check out our Code of Conduct.













paolo galli is a new contributor. Be nice, and check out our Code of Conduct.












paolo galli is a new contributor. Be nice, and check out our Code of Conduct.















 


draft saved


draft discarded














StackExchange.ready(
function () {
StackExchange.openid.initPostLogin('.new-post-login', 'https%3a%2f%2fmath.stackexchange.com%2fquestions%2f3004600%2fexponents-of-ec-primes-ending-with-digit-7%23new-answer', 'question_page');
}
);

Post as a guest















Required, but never shown





















































Required, but never shown














Required, but never shown












Required, but never shown







Required, but never shown

































Required, but never shown














Required, but never shown












Required, but never shown







Required, but never shown







Popular posts from this blog

Can a sorcerer learn a 5th-level spell early by creating spell slots using the Font of Magic feature?

Does disintegrating a polymorphed enemy still kill it after the 2018 errata?

A Topological Invariant for $pi_3(U(n))$